Đến nội dung

shinichigl

shinichigl

Đăng ký: 18-05-2013
Offline Đăng nhập: 19-12-2017 - 00:10
***--

#688685 $x^{y}=(y+1)^{x}$

Gửi bởi shinichigl trong 26-07-2017 - 10:58

Đặt $\sqrt{y+1}=b$; $\sqrt{-4x^{2}+18x-20}=a$

Từ đó ta có: $b> 0$; $a\geq 0$; $2\leq x\leq 2.5$

Từ phương trình đầu tiên ta có:

$1+a=b-\frac{4}{a^{2}+4}$

$\Leftrightarrow b=\frac{a^{3}+a^{2}+4a+8}{a^{2}+4}$

$\Rightarrow b\geq \frac{8}{4}=2$ hay $b^{2}\geq 4$

(Do hàm số $f(a)=\frac{a^{3}+a^{2}+4a+8}{a^{2}+4}$ là hàm đồng biến trên tập xác định (tính đạo hàm ra sẽ thấy))

Từ phương trình thứ hai ta có:

$x^{b^{2}}=\left ( b^{2} \right )^{x}$

$\Leftrightarrow ln\left ( x^{b^{2}} \right )=ln\left ( \left ( b^{2} \right )^{x} \right )$

$\Leftrightarrow b^{2}ln(x)=xln(b^{2})$

$\Leftrightarrow \frac{ln(x)}{x}=\frac{ln\left ( b^{2} \right )}{b^{2}}$

Xét $f(t)=\frac{ln(t)}{t}$ $(t> 0)$

$f'(t)=\frac{1-ln(t)}{t^{2}}$

Từ đó ta có:

Với $0<t<e$ thì $f(t)$ đồng biến

Với $t>e$ thì $f(t)$ nghịch biến

Suy ra: 

$\frac{ln(x)}{x}\geq \frac{ln(2)}{2}$ và $\frac{ln(b^{2})}{b^{2}}\leq  \frac{ln(4)}{4}=\frac{ln(2)}{2}$

Đẳng thức xảy ra khi: $b=2$; $x=2$ hay $y=3$; $x=2$ (*)

Thay (*) vào phương trình thứ nhất ta thấy thõa mãn

Vậy hệ phương trình có 1 nghiệm là $\left (x;y \right )=\left ( 2;3 \right )$




#688575 $2(\sqrt{1-5x}-\sqrt{x}-\sqrt{x-...

Gửi bởi shinichigl trong 24-07-2017 - 22:28

Điều kiện: $0\leq 0\leq \frac{1}{5}$ (1)

$2\left ( \sqrt{1-5x}-\sqrt{x} -\sqrt{x-x^{2}}\right )=x-1$

$\Leftrightarrow 2\left ( \sqrt{1-5x} -\sqrt{x-x^{2}}\right )=2\sqrt{x}-\left ( 1-x \right )$

$\Leftrightarrow 2\frac{x^{2}-6x+1}{\sqrt{1-5x}+\sqrt{x-x^{2}}}=(-1)\frac{x^{2}-6x+1}{2\sqrt{x}+\left ( 1-x \right )}$

$\Leftrightarrow x^{2}-6x+1=0$

(Vì phương trình $\frac{2}{\sqrt{1-5x}+\sqrt{x-x^{2}}}=\frac{-1}{2\sqrt{x}+\left ( 1-x \right )}$ vô nghiệm (VT>0 và VP<0))

$\Leftrightarrow x=3-2\sqrt{2}$ (do (1))




#688464 GPT: $\sqrt[3]{2x^3+6}=x+\sqrt{x^2-3x+3}...

Gửi bởi shinichigl trong 23-07-2017 - 21:50

Đặt $\sqrt{x^{2}-3x+3}=b$ (1) ($b\geq 0$)

Ta có hệ phương trình

$\left\{\begin{matrix} 2x^{3}+6=\left ( x+b \right )^{3}\\ x^{2}-3x+3=b^{2} \end{matrix}\right.$

$\Leftrightarrow \left\{\begin{matrix} \left ( x-b \right )^{3}+6=6xb^{2}\\ x^{2}-3x+3=b^{2} \end{matrix}\right.$
$\Leftrightarrow \left\{\begin{matrix} \left ( x-b \right )^{3}+6=6x\left ( x^{2}-3x+3 \right )\\ x^{2}-3x+3=b^{2} \end{matrix}\right.$
$\Leftrightarrow \left\{\begin{matrix} \left ( x-b \right )^{3}=6\left ( x-1 \right )^{3}\\ x^{2}-3x+3=b^{2} \end{matrix}\right.$
$\Rightarrow b=\left ( 1-\sqrt[3]{6} \right )x+\sqrt[3]{6}$ (2)
Thay (2) vào (1) ta tìm được 2 nghiệm là
$x_{1}=1;x_{2}=\frac{\sqrt[3]{36}-3}{\sqrt[3]{36}-2\sqrt[3]{6}}$



#687653 $\sum \frac{a}{b}\geq \frac{3}{2}\left ( a+b+c-...

Gửi bởi shinichigl trong 15-07-2017 - 22:17

Trong 3 số (a-1), (b-1), (c-1) luôn luôn có 2 số cùng không âm hoặc không dương. Giả sử (a-1) và (b-1) cùng không âm hoặc cùng không dương
Khi đó ta có: (a-1)(b-1)$\geq 0$ (*)

Từ đề bài: $a=\frac{1}{bc}$. Ta cần chứng minh $\frac{1}{b^{2}c}+\frac{b}{c}+bc^{2}\geq \frac{3}{2}\left ( \frac{1}{bc}+b+c-1 \right )$

Ta lại có:

$\frac{1}{b^{2}c}+\frac{b}{c}+bc^{2}=\left ( \frac{1}{2b^{2}c}+\frac{b}{c} \right )+\left ( \frac{1}{2b^{2}c}+bc^{2} \right )$

$\geq\frac{3}{2} \left ( \sqrt[3]{\frac{1}{b^{2}c}.\left ( \frac{b}{c} \right )^{2}}+\sqrt[3]{\frac{1}{b^{2}c}.\left ( bc^{2} \right )^{2}} \right )$

$=\frac{3}{2}\left ( \frac{1}{c}+c \right )$

Mặt khác: $\frac{1}{c}+c\geq \frac{1}{bc}+b+c-1\Leftrightarrow \frac{1}{c}+1-\frac{1}{bc}-b\geq 0\Leftrightarrow \left ( b-1 \right )\left ( \frac{1}{bc}-1 \right )\geq 0\Leftrightarrow \left ( b-1 \right )\left ( a-1 \right )\geq 0$ Đúng theo (*)
Vậy bđt được chứng minh



#551488 Đề thi Olympic 30/4 chính thức lần thứ XXI năm 2015 (lớp 11)

Gửi bởi shinichigl trong 05-04-2015 - 00:45

Bài 6 (3 điểm)

Tại ba đỉnh $A,B,C$ của một tam giác $ABC$, người ta viết các số $a,b,c$. Người ta thực hiện phép biến đổi sau : Nếu mỗi bộ trước là $(x,y,z)$ thì sau đó ta thay bởi bộ $(x+y-2z,y+z-2x,z+x-2y)$. Chứng minh rằng sau một số lần biến đổi sẽ tồn tại một bộ ba số mà ít nhất một trong ba số của nó không nhỏ hơn $2015$.

 

Bài 6 (Mình nghĩ cần phải thêm điều kiện ít nhất trong 3 số $a$, $b$, $c$ phải có một số khác $0$)

Sau lần biến đổi thứ nhất thì tổng của 3 số $a$, $b$, $c$ bằng $0$ nên ta có $a_{n}+b_{n}+c_{n}=0,n\geq 1$

($n$ là số lần biến đổi của bộ $\left ( a,b,c \right )$)

Ta xét tổng $S_{n}=a_{n}^{2}+b_{n}^{2}+c_{n}^{2}$

Khi đó $S_{n+1}=\left ( a_{n}+b_{n}-2c_{n} \right )_{n}^{2}+\left ( b_{n}+c_{n}-2a_{n} \right )^{2}+\left ( c_{n}+a_{n}-2b_{n} \right )^{2}=6S_{n}-6\left ( a_{n}b_{n}+b_{n}c_{n}+c_{n}a_{n} \right )$

Mặt khác $a_{n}+b_{n}+c_{n}=0$ nên $a_{n}^{2}+b_{n}^{2}+c_{n}^{2}=-2\left ( a_{n}b_{n}+b_{n}c_{n}+c_{n}a_{n} \right )$

Suy ra $S_{n+1}=3S_{n}=3^{n}S_{1}$

Ta có $\left ( \left | a_{n} \right |+\left | b_{n} \right |+\left | c_{n} \right | \right )^{2}> a_{n}^{2}+b_{n}^{2}+c_{n}^{2}$

Suy ra $\left | a_{n} \right |+\left | b_{n} \right |+\left | c_{n} \right |> 3^{\frac{n-1}{2}}\sqrt{S_{1}}$

(do trong 3 số $a$, $b$, $c$ phải có ít nhất một số khác $0$ nên $S_{1}>0$)

Giả sử $\left | a_{n} \right |$ là số lớn nhất trong 3 số $\left | a_{n} \right |$, $\left | b_{n} \right |$, $\left | c_{n} \right |$

Khi đó $\left | a_{n} \right |> 3^{\frac{n-3}{2}}\sqrt{S_{1}}$. Nếu $a_{n}>0$ thì $a_{n}> 3^{\frac{n-3}{2}}\sqrt{S_{1}}>\frac{3^{\frac{n-3}{2}}\sqrt{S_{1}}}{2}$

Nếu $a_{n}<0$ thì $a_{n}<- 3^{\frac{n-3}{2}}\sqrt{S_{1}}$ hay $b_{n}+c_{n}> 3^{\frac{n-3}{2}}\sqrt{S_{1}}$

Suy ra một trong 2 số $b_{n}$, $c_{n}$ phải lớn hơn $\frac{3^{\frac{n-3}{2}}\sqrt{S_{1}}}{2}$

Vậy một trong 3 số $a_{n}$, $b_{n}$, $c_{n}$ phải lớn hơn $\frac{3^{\frac{n-3}{2}}\sqrt{S_{1}}}{2}$.

Từ đó với $n$ đủ lớn thì $\frac{3^{\frac{n-3}{2}}\sqrt{S_{1}}}{2}\geq 2015$

Vậy ta có điều phải chứng minh




#549749 $\sum\frac{x}{x^2+1}\leq \frac...

Gửi bởi shinichigl trong 27-03-2015 - 16:11

Cách khác:

Áp dụng $AM-GM$, ta có:

$x^2+\frac{1}{9}\geq 2\sqrt{x^2.\frac{1}{9}}=\frac{2}{3}x$

Do đó, $\sum\frac{x}{x^2+1}\leq\sum\frac{x}{\frac{2}{3}x+\frac{8}{9}}$

$\Leftrightarrow\sum\frac{x}{x^2+1}\leq\sum\frac{9x}{6x+8}$

$\Leftrightarrow\frac{2}{3}\sum\frac{x}{x^2+1}\leq\sum\frac{6x}{6x+8}=\sum 1-\frac{8}{6x+8}\leq 3-8\frac{9}{\sum 6x+8}=3-\frac{72}{6(x+y+z)+24}$

$\Leftrightarrow\sum\frac{x}{x^2+1}\leq\frac{3}{2}(3-\frac{72}{30})=\frac{9}{10}$

x,y,z là số thực nên đâu có chỗ này đâu bạn




#546340 $ n\mid k^{a}-k^{b} $

Gửi bởi shinichigl trong 26-02-2015 - 17:42

Ta có $k^{a}-k^{b}\vdots n,\forall k$ nên $k^{\left | a-b \right |}-1\vdots n,\forall k$.Từ đó ta cũng có $\left (2k  \right )^{\left | a-b \right |}-1\vdots n,\forall k$.

Suy ra $2^{\left | a-b \right |}-1\vdots n$. Do $0\leq a,b\leq 9$ nên $0\leq \left | a-b \right |\leq 9$.

Từ đó lần lượt thay các giá trị của $\left | a-b \right |$ ta sẽ tìm được nghiệm.

Vậy các nghiệm thõa mãn đề bài là $15$ và $\overline{aa}$.




#545877 Chứng minh $(a^2+b^2, c^2+d^2)>1$

Gửi bởi shinichigl trong 24-02-2015 - 16:31

Cho các số nguyên dương a, b, c, d sao cho $(ac+bd) \vdots (a^2+b^2)$.  Chứng minh $(a^2+b^2, c^2+d^2)>1$ 

Từ giả thiết ta có $a\left ( ac+bd \right )\vdots \left ( a^{2}+b^{2} \right )$. Suy ra $c\left ( a^{2}+b^{2} \right )+b\left ( ad-bc \right )\vdots \left ( a^{2}+b^{2} \right )$.

Như vậy $b\left ( ad-bc \right )\vdots \left ( a^{2}+b^{2} \right )$. Suy ra $b\left ( adc-bc^{2} \right )\vdots \left ( a^{2}+b^{2} \right )$ (1).

Cũng từ giả thiết đề bài ta có $b\left ( adc+bd^{2} \right )\vdots \left ( a^{2}+b^{2} \right )$ (2).

Từ (1) và (2) ta có $b^{2}\left ( c^{2}+d^{2} \right )\vdots \left ( a^{2}+b^{2} \right )$.

Nếu $\left ( a^{2}+b^{2};c^{2}+d^{2} \right )=1$ thì $b^{2}\vdots \left ( a^{2}+b^{2} \right )$ (vô lý).

Vậy $\left ( a^{2}+b^{2};c^{2}+d^{2} \right )>1$.

Nhận xét: Nếu $\left ( a;b \right )=1$ thì $\left ( c^{2}+d^{2} \right )\vdots \left ( a^{2}+b^{2} \right )$.




#545857 $a^n-1$ không chia hết cho $n$

Gửi bởi shinichigl trong 24-02-2015 - 14:40

Cho $n$ là số nguyên dương lẻ và lớn hơn $1$. Chứng minh rằng với mọi số $a=2^k+1$ với $k$ là số nguyên dương thì 

$a^n-1$ không chia hết cho $n$

ch: chia hết cho; kch: không chia hết cho.

Giả sử $n$ là số nguyên dương lẻ nhỏ nhất thõa mãn $a^{n}-1$ ch $n$.

Nếu $\left ( a;n \right )=f>1$ thì $a^{n}-1$ kch $f$. Suy ra $a^{n}-1$ kch $n$.

Vậy $\left ( a;n \right )=1$. Theo tiêu chuẩn Euler ta có $a^{\varphi \left ( n \right )}-1$ ch $n$.

Mặt khác $\left (a^{n}-1;a^{\varphi \left ( n \right )}-1  \right )=a^{d}-1$, với $\left ( n;\varphi \left ( n \right ) \right )=d$.

Từ đó $a^{d}-1$ ch $n$. Nếu $d=1$ thì $a-1=2^{k}$ kch $n$ (do $n$ là số lẻ).

Vậy $d>1$. Suy ra $a^{d}-1$ ch $d$ (do $\left ( n;\varphi \left ( n \right ) \right )=d$).

Do $\left ( n;\varphi \left ( n \right ) \right )=d$ nên $1<d\leq \varphi \left ( n \right )<n$

Điều này vô lý với định nghĩa $n$ ban đầu.

Vậy $a^{n}-1$ kch $n$

Chú ý $\left ( a^{m}-1;a^{n}-1 \right )=a^{d}-1$, với $d=\left ( m;n \right )$

Thật vậy,

Giả sử $m>n$ và $m=qn+r$.

Ta có $\left ( a^{m}-1;a^{n}-1 \right )=\left ( a^{qn}-1+a^{qn}\left ( a^{r}-1 \right );a^{n}-1 \right )=\left ( a^{r}-1;a^{n}-1 \right )$

(do $a^{m}-1$ và $a^{n}-1$ kch $a$).

Từ đó ta có $\left ( m;n \right )=\left ( n;r \right )\Leftrightarrow \left ( a^{m}-1;a^{n}-1 \right )=\left ( a^{n}-1;a^{r}-1 \right )$.

Vậy theo thuật chia Euclide thì ta có điều phải chứng minh.




#545853 Đề thi chọn đội tuyển HSG QG Hà Nội năm học 2014-2015

Gửi bởi shinichigl trong 24-02-2015 - 14:10

Đề thi chọn đột tuyển Hà Nội vòng 2 năm học 2014-2015

Bài 2

Nếu tồn tại $y_{0}$ sao cho $f\left (y_{0}  \right )>1$ thì thay $x$ bởi $\frac{y_{0}}{f\left ( y_{0} \right )-1}$ ta có

$f\left ( \frac{y_{0}f\left ( y_{0} \right )}{f\left ( y_{0} \right )-1} \right )f\left ( y_{0} \right )=f\left ( \frac{y_{0}f\left ( y_{0} \right )}{f\left ( y_{0} \right )-1} \right )$. Suy ra $f\left ( y_{0} \right )=1$ (vô lý). Vậy $f\left ( y \right )\leq 1,\forall y>0$

Từ đó ta có $f\left ( y \right )\geq f\left ( x+y \right ),\forall x>0,y>0$. Suy ra $f$ là hàm không tăng

Nếu tồn tại $y_{1}$ sao cho $f\left (y_{1}  \right )=1$ thì thay $y$ bởi $y_{1}$ ta có $f\left ( x \right )=f\left ( x+y_{1} \right ),\forall x>0$

Do $f$ là hàm không tăng và tuần hoàn nên $f$ là hàm hằng. Vậy $f\left ( y \right )=1,\forall y>0$

Ta xét $f\left ( y \right )< 1,\forall y>0$. Suy ra $f$ nghịch biến hay $f$ là hàm đơn ánh.

Thay $x$ bởi $\frac{x}{f\left ( y \right )}$ ta có $f\left ( x \right )f\left ( y \right )=f\left ( \frac{x}{f\left ( y \right )}+y \right ),\forall x>0,y>0$

Hoán đổi vị trí $x$ và $y$ ta được $f\left (\frac{x}{f\left ( y \right )}+y  \right )=f\left ( \frac{y}{f\left ( x \right )}+x \right ),\forall x>0,y>0$

Suy ra $\frac{x}{f\left ( y \right )}+y=\frac{y}{f\left ( x \right )}+x,\forall x>0,y>0$

Như vậy $\frac{1-f\left ( y \right )}{yf\left ( y \right )}=k,\forall y>0$ ($k$ là hằng số, $k\neq 0$)

Suy ra $f\left ( y \right )=\frac{1}{1+ky},\forall y>0$

Thử lại 2 hàm trên, ta thấy đều thõa mãn đề bài.

Vậy $f\left ( y \right )\equiv 1$, $f\left ( y \right )\equiv \frac{1}{1+ky}$




#541175 VMO 2015

Gửi bởi shinichigl trong 18-01-2015 - 13:15

 

NGÀY 2
Bài 5: (7,0 điểm) Cho $(f_n(x))$ là dãy đa thức xác định bởi:
$f_0(x)=2,f_1(x)=3x,f_n(x)=3xf_{n-1}(x)+(1-x-2x^2)f_{n-2}(x)$ với mọi $n\ge 2$. 
Tìm tất cả các số nguyên dương $n$ để $f_n(x)$ chia hết cho đa thức $x^3-x^2+x$.

Áp dụng phương trình sai phân bậc hai, ta tìm được công thức tổng quát: $f_{n}(x)=(2x-1)^{n}+(x+1)^{n}$ (*)

Thay $x=5$ vào (*) ta được $f_{n}(5)=9^{n}+6^{n}$

Ta sẽ tìm $n$ để $f_{n}(5)=9^{n}+6^{n}$ chia hết cho $5^{3}-5^{2}+5=105$

Ta có $9^{n}+6^{n}=3^{n}\left ( 3^{n}+2^{n} \right )$ và $105=3.5.7$

Suy ra ta chỉ cần tìm $n$ sao cho $3^{n}+2^{n}$ chia hết cho $7$

Đặt $n=6k+r$ ($r\in \left \{ 0,1,2,3,4,5 \right \}$)

Từ đó ta có $3^{n}+2^{n}=3^{6k+r}+2^{6k+r}=729^{k}.3^{r}+64^{k}.3^{r}$

Suy ra $3^{n}+2^{n}\equiv 3^{r}+2^{r}(mod7)$

Lần lượt thay $r= 0,1,2,3,4,5$ ta thấy chỉ có $r=3$ thõa mãn $3^{n}+2^{n}\equiv 0(mod7)$

Tiếp theo ta sẽ chứng minh $n=6k+3$ thõa mãn (bằng quy nạp theo $k$)

Thật vậy,

Với $k=0$ ($n=3$) ta có $f_{n}=f_{3}=9x^{3}-9x^{2}+9x$ chia hết cho $x^{3}-x^{2}+x$

Giả sử $k=a$ thõa mãn, ta sẽ chứng minh $k=a+1$ cũng thõa mãn

Ta có 

$(2x-1)^{6a+9}+(x+1)^{6a+9}=\left [ (2x-1)^{6a+3}+(x+1)^{6a+3} \right ]\left [ (2x-1)^{6}+(x+1)^{6} \right ]-(2x-1)^{6}(x+1)^{6}\left [ (2x-1)^{6a-3}+(x+1)^{6a-3} \right ]$

chia hết cho $x^{3}-x^{2}+x$

Vậy theo nguyên lý quy nạp ta có $n=6k+3$, với mọi $k$ nguyên không âm, thõa mãn đề bài




#541172 $y^3=3x^2+3x+7$

Gửi bởi shinichigl trong 18-01-2015 - 12:04

Cho $x,y \in \mathbb{Z}$.tìm $x,y$ tm:

$y^3=3x^2+3x+7$

Từ giả thiết ta suy ra $y\equiv 1(mod3)$. Đặt $y=3k+1$

Từ đó ta có $(3k+1)^{3}=3x^{2}+3x+7\Leftrightarrow 9k^{3}+9k^{2}+3k=x^{2}+x+2$

Suy ra $x^{2}+x+2\equiv 0(mod3)$

Mặt khác $x^{2}+x+2\equiv 0(mod3)$

          $\Leftrightarrow 4x^{2}+4x+8\equiv 0(mod3)$

          $\Leftrightarrow (2x+1)^{2}+7\equiv 0(mod3)$

          $\Leftrightarrow (2x+1)^{2}+1\equiv 0(mod3)$ (*)

Ta có $(2x+1)^{2}\equiv 0;1(mod3)$ (do một số chính phương thì chia 3 dư 0 hoặc 1)

      $\Leftrightarrow (2x+1)^{2}+1\equiv 1;2(mod3)$. Mâu thuẫn với (*)

Vậy phương trình trên không có nghiệm nguyên trên $\mathbb{Z}$




#535485 $f(x+f(x)+2y)=2x+2f(f(y))$

Gửi bởi shinichigl trong 30-11-2014 - 10:29

Tìm hàm $f:\mathbb{Q}\rightarrow \mathbb{Q}$ thỏa mãn: $f(x+f(x)+2y)=2x+2f(f(y))$ (*)

Trong (*) thay $x=y$ ta được: $f(f(0))=2f((0)) \Rightarrow f(f(0))=0$

Trong (*) thay $y=0$ ta được: $f(x+f(x))=2x,\forall x\in \mathbb{Q}$ (1) $\Rightarrow$ $f$ toàn ánh

Do $f$ toàn ánh nên tồn tại $a$ sao cho $f(a)=0$

Thay $x=a$ vào (1) ta được: $f(a)=2a \Rightarrow f(0)=0$

Trong (*) thay $x=0$ ta được: $f(2y)=2f(f(y)), \forall y \in \mathbb{Q}$ (2)

Trong (1) cho $x=1$ ta được $f(1+f(1))=2$

Do f toàn ánh nên tồn tại $c$ sao cho $f\left ( c \right )=1+f(1)$

Từ (*) và (2) ta có: $f(x+f(x)+2y)=2x+f(2y), \forall x,y \in \mathbb{Q}$

$\Rightarrow f(x+f(x)+y)=2x+f(y), \forall x,y \in \mathbb{Q}$ (3)

Trong (3) thay $x=1$ và $y=-1-f(1)$ ta được: $f(-1-f(1))=-2$

Trong (3) thay $x=c$ và $y=-1-f(1)$ ta được: $f\left ( c \right )=2c-2 \Rightarrow f(1)+3=2c$

Trong (2) thay $y=c$ ta được: $f(2c)=4$

Trong (*) thay $x=y=1$ ta được: $f(3+f(1))=2+2f(f(1)) \Rightarrow f(f(1))=1$

Trong (1) thay $x=f(1)$ ta được: $f(f(1)+1)=2f(1) \Rightarrow f(1)=1$

Trong (3) thay $x=1$ ta được: $f(2+y)=2+f(y),\forall y\in \mathbb{Q}$ (4)

Từ (4) bằng quy nạp ta chứng minh được: $f(x)=x,\forall x\in \mathbb{Z}$

Từ (3) bằng quy nạp ta chứng minh được: $f\left ( n\left ( x+f(x) \right ) \right )=nf\left ( x+f(x) \right )=2nx,\forall n\in \mathbb{Z},x\in \mathbb{Q}$ (5)

Trong (5) thay $x=\frac{1}{2n}$ ta được: $f\left ( n\left ( \frac{1}{2n}+f\left ( \frac{1}{2n} \right ) \right ) \right )=1$

Đặt $n\left ( \frac{1}{2n}+f\left ( \frac{1}{2n} \right ) \right )=b$

Trong (3) thay $x=b$ và $y=-f(b)$ ta được: $f\left ( b+f(b)-f(b) \right )=2b+f(-f(b))\Rightarrow b=1$

$\Rightarrow f\left ( \frac{1}{2n} \right )=\frac{1}{2n},\forall n\in \mathbb{Z}^{*}$

Trong (5) thay $x$ bởi $\frac{1}{2m}$ ta được: $f\left ( n\left ( \frac{1}{2m}+f\left ( \frac{1}{2m} \right ) \right ) \right )=\frac{n}{m},\forall n\in \mathbb{Z},m\in \mathbb{Z}^{*}$

$\Rightarrow f\left ( \frac{n}{m} \right )=\frac{n}{m},\forall n\in \mathbb{Z},m\in \mathbb{Z}^{*}$

$\Rightarrow f(x)=x,\forall x\in \mathbb{Q}$




#533852 $f(yf(x+y)+f(x))= 4x+2yf(x+y)$

Gửi bởi shinichigl trong 20-11-2014 - 00:48

$f:R\rightarrow R$ thoả mãn $f(yf(x+y)+f(x))= 4x+2yf(x+y) (1)$ 

Trong (1) cho $y=0$ ta được $f(f(x))=4x,\forall x\in \mathbb{R}$ (2), suy ra $f$ song ánh

Từ đó tồn tại duy nhất $c$ sao cho $f\left ( c \right )=2$

Trong (1) thay $y$ bởi $c-x$ ta có

$f((c-x)f\left ( c \right )+f(x))=4x+2(c-x)f\left ( c \right ),\forall x\in \mathbb{R}
\Rightarrow f(2c-2x+f(x))=4c,\forall x\in \mathbb{R}$

Trong (2) cho $x=c$ ta được $f(f\left ( c \right ))=4c=f(2)$

suy ra $2c-2x+f(x)=2,\forall x\in \mathbb{R}
\Rightarrow f(x)=2x+2-2c,\forall x\in \mathbb{R}$

Từ đó ta tìm được $f(x)=2x,\forall x\in \mathbb{R}
$




#529423 Chọn đội tuyển QG tỉnh Gia Lai 2014-2015

Gửi bởi shinichigl trong 18-10-2014 - 20:47

Câu 5:

Ta sẽ chứng minh $\mathbb{N}^{*}\subset A$

Điều này tương đương với việc chứng minh $\forall x\in \mathbb{N}^{*}$ thì $x\in A$

Theo giả thiết $\left \{ 1;2;3 \right \}\in A$

Do $2;3\in A$ nên $1+2.3=7\in A$

Do $2;7\in A$ nên $1+2.7=15\in A$ $\Rightarrow 5\in A$

Do $5;3\in A$ nên $1+5.3=16\in A$ $\Rightarrow 4\in A$

Do $5;7\in A$ nên $1+5.7=36\in A$ $\Rightarrow 6\in A$

Từ đó ta có $\left \{ 1;2;3;4;5;6;7 \right \}\in A$

Giả sử $k\in A$

Trường hợp 1: $k$ chẵn

Khi đó ta có $\frac{k}{2}\in \mathbb{N}^{*}$

Mà $\frac{k}{2}<k$ nên $\frac{k}{2}\in A$

Suy ra $1+2.\frac{k}{2}=k+1\in A$

Trường hợp 2: $k$ lẻ

Khi đó ta có $\frac{k+1}{2}\in \mathbb{N}^{*}$

Mà $\frac{k+1}{2}<k$ nên $\frac{k+1}{2}\in A$

Suy ra $1+2.\frac{k+1}{2}=k+2\in A$

Suy ra $1+\left ( k+2 \right )k=\left ( k+1 \right )^{2}\in A$ $\Rightarrow k+1\in A$

Từ hai trường hợp trên ta có $k+1\in A$

Vậy theo nguyên lí quy nạp ta chứng minh được $\forall x\in \mathbb{N}^{*}$ thì $x\in A$

Từ đó ta có $\mathbb{N}^{*}\subset A$

Mà theo giả thiết thì ta có $A\subset \mathbb{N}^{*}$

Suy ra $A=\mathbb{N}^{*}$

Vậy $2013^{2014}\in A$

 

@supermember: giỏi quá (clap)